Solve recurrence relation for non trivial base case Announcing the arrival of Valued Associate #679: Cesar Manara Planned maintenance scheduled April 17/18, 2019 at 00:00UTC (8:00pm US/Eastern)Solving recurrence relation with unrolling techniqueSolving recurrence relation of algorithm complexity?How do I solve the following recurrence?Solving this recurrence relationHow to solve the recurrence relation $T(n) = T(lceil n/2rceil) + T(lfloor n/2rfloor) + 2$Solving recurrence relationRecurrence equation - floor problemSolving recurrence equation with floor and ceil functionsShow the recursive sequence is increasingSolving this recurrence relation problem

Area of a 2D convex hull

How can you insert a "times/divide" symbol similar to the "plus/minus" (±) one?

Can a non-EU citizen traveling with me come with me through the EU passport line?

How does modal jazz use chord progressions?

3 doors, three guards, one stone

What computer would be fastest for Mathematica Home Edition?

Is drag coefficient lowest at zero angle of attack?

If A makes B more likely then B makes A more likely"

What loss function to use when labels are probabilities?

Why does tar appear to skip file contents when output file is /dev/null?

Antler Helmet: Can it work?

How is simplicity better than precision and clarity in prose?

Is it possible to ask for a hotel room without minibar/extra services?

Was credit for the black hole image misattributed?

Blender game recording at the wrong time

Why use gamma over alpha radiation?

Is there a service that would inform me whenever a new direct route is scheduled from a given airport?

Typsetting diagram chases (with TikZ?)

When is phishing education going too far?

Can a zero nonce be safely used with AES-GCM if the key is random and never used again?

Stop battery usage [Ubuntu 18]

What do you call the holes in a flute?

What did Darwin mean by 'squib' here?

Did the new image of black hole confirm the general theory of relativity?



Solve recurrence relation for non trivial base case



Announcing the arrival of Valued Associate #679: Cesar Manara
Planned maintenance scheduled April 17/18, 2019 at 00:00UTC (8:00pm US/Eastern)Solving recurrence relation with unrolling techniqueSolving recurrence relation of algorithm complexity?How do I solve the following recurrence?Solving this recurrence relationHow to solve the recurrence relation $T(n) = T(lceil n/2rceil) + T(lfloor n/2rfloor) + 2$Solving recurrence relationRecurrence equation - floor problemSolving recurrence equation with floor and ceil functionsShow the recursive sequence is increasingSolving this recurrence relation problem










1












$begingroup$


As per the title, I'm having some trouble to solve the recurrence equation



Edited



$$ T(N) = 2T left(leftlceil fracN+12 rightrceilright) + 2T left(leftlfloor fracN+12 rightrfloorright)$$
which is true for $N > 4$. I have two base cases, $T(3) = 6$ and $T(4) = 18$, but I'm stuck on how to proceed.










share|cite|improve this question











$endgroup$











  • $begingroup$
    Your base cases are contradictory. $T(3)=4T(2)$ and $T(4)=4T(2)$ so $T(3)=T(4)$ but this is not true.
    $endgroup$
    – Peter Foreman
    Mar 25 at 21:20






  • 1




    $begingroup$
    Yeah, sorry it's fine now.
    $endgroup$
    – Peter Foreman
    Mar 25 at 22:25






  • 2




    $begingroup$
    You mentioned $n^2 ln n$, are you interested in the asymptotic behavior of $T(n)$? It is in fact $Theta(n^2)$ by the master theorem.
    $endgroup$
    – Maxim
    Mar 27 at 21:48










  • $begingroup$
    @Maxim thanks, it is surely better than nothing. But it could be great to also show the exact solution (constants throws in) or at least an approximation of it. Of course I'm also interested in the explanation (which in the case of master theorem is obvious).
    $endgroup$
    – tigerjack89
    Mar 28 at 6:41















1












$begingroup$


As per the title, I'm having some trouble to solve the recurrence equation



Edited



$$ T(N) = 2T left(leftlceil fracN+12 rightrceilright) + 2T left(leftlfloor fracN+12 rightrfloorright)$$
which is true for $N > 4$. I have two base cases, $T(3) = 6$ and $T(4) = 18$, but I'm stuck on how to proceed.










share|cite|improve this question











$endgroup$











  • $begingroup$
    Your base cases are contradictory. $T(3)=4T(2)$ and $T(4)=4T(2)$ so $T(3)=T(4)$ but this is not true.
    $endgroup$
    – Peter Foreman
    Mar 25 at 21:20






  • 1




    $begingroup$
    Yeah, sorry it's fine now.
    $endgroup$
    – Peter Foreman
    Mar 25 at 22:25






  • 2




    $begingroup$
    You mentioned $n^2 ln n$, are you interested in the asymptotic behavior of $T(n)$? It is in fact $Theta(n^2)$ by the master theorem.
    $endgroup$
    – Maxim
    Mar 27 at 21:48










  • $begingroup$
    @Maxim thanks, it is surely better than nothing. But it could be great to also show the exact solution (constants throws in) or at least an approximation of it. Of course I'm also interested in the explanation (which in the case of master theorem is obvious).
    $endgroup$
    – tigerjack89
    Mar 28 at 6:41













1












1








1


1



$begingroup$


As per the title, I'm having some trouble to solve the recurrence equation



Edited



$$ T(N) = 2T left(leftlceil fracN+12 rightrceilright) + 2T left(leftlfloor fracN+12 rightrfloorright)$$
which is true for $N > 4$. I have two base cases, $T(3) = 6$ and $T(4) = 18$, but I'm stuck on how to proceed.










share|cite|improve this question











$endgroup$




As per the title, I'm having some trouble to solve the recurrence equation



Edited



$$ T(N) = 2T left(leftlceil fracN+12 rightrceilright) + 2T left(leftlfloor fracN+12 rightrfloorright)$$
which is true for $N > 4$. I have two base cases, $T(3) = 6$ and $T(4) = 18$, but I'm stuck on how to proceed.







recurrence-relations






share|cite|improve this question















share|cite|improve this question













share|cite|improve this question




share|cite|improve this question








edited Mar 26 at 7:49







tigerjack89

















asked Mar 25 at 21:04









tigerjack89tigerjack89

889




889











  • $begingroup$
    Your base cases are contradictory. $T(3)=4T(2)$ and $T(4)=4T(2)$ so $T(3)=T(4)$ but this is not true.
    $endgroup$
    – Peter Foreman
    Mar 25 at 21:20






  • 1




    $begingroup$
    Yeah, sorry it's fine now.
    $endgroup$
    – Peter Foreman
    Mar 25 at 22:25






  • 2




    $begingroup$
    You mentioned $n^2 ln n$, are you interested in the asymptotic behavior of $T(n)$? It is in fact $Theta(n^2)$ by the master theorem.
    $endgroup$
    – Maxim
    Mar 27 at 21:48










  • $begingroup$
    @Maxim thanks, it is surely better than nothing. But it could be great to also show the exact solution (constants throws in) or at least an approximation of it. Of course I'm also interested in the explanation (which in the case of master theorem is obvious).
    $endgroup$
    – tigerjack89
    Mar 28 at 6:41
















  • $begingroup$
    Your base cases are contradictory. $T(3)=4T(2)$ and $T(4)=4T(2)$ so $T(3)=T(4)$ but this is not true.
    $endgroup$
    – Peter Foreman
    Mar 25 at 21:20






  • 1




    $begingroup$
    Yeah, sorry it's fine now.
    $endgroup$
    – Peter Foreman
    Mar 25 at 22:25






  • 2




    $begingroup$
    You mentioned $n^2 ln n$, are you interested in the asymptotic behavior of $T(n)$? It is in fact $Theta(n^2)$ by the master theorem.
    $endgroup$
    – Maxim
    Mar 27 at 21:48










  • $begingroup$
    @Maxim thanks, it is surely better than nothing. But it could be great to also show the exact solution (constants throws in) or at least an approximation of it. Of course I'm also interested in the explanation (which in the case of master theorem is obvious).
    $endgroup$
    – tigerjack89
    Mar 28 at 6:41















$begingroup$
Your base cases are contradictory. $T(3)=4T(2)$ and $T(4)=4T(2)$ so $T(3)=T(4)$ but this is not true.
$endgroup$
– Peter Foreman
Mar 25 at 21:20




$begingroup$
Your base cases are contradictory. $T(3)=4T(2)$ and $T(4)=4T(2)$ so $T(3)=T(4)$ but this is not true.
$endgroup$
– Peter Foreman
Mar 25 at 21:20




1




1




$begingroup$
Yeah, sorry it's fine now.
$endgroup$
– Peter Foreman
Mar 25 at 22:25




$begingroup$
Yeah, sorry it's fine now.
$endgroup$
– Peter Foreman
Mar 25 at 22:25




2




2




$begingroup$
You mentioned $n^2 ln n$, are you interested in the asymptotic behavior of $T(n)$? It is in fact $Theta(n^2)$ by the master theorem.
$endgroup$
– Maxim
Mar 27 at 21:48




$begingroup$
You mentioned $n^2 ln n$, are you interested in the asymptotic behavior of $T(n)$? It is in fact $Theta(n^2)$ by the master theorem.
$endgroup$
– Maxim
Mar 27 at 21:48












$begingroup$
@Maxim thanks, it is surely better than nothing. But it could be great to also show the exact solution (constants throws in) or at least an approximation of it. Of course I'm also interested in the explanation (which in the case of master theorem is obvious).
$endgroup$
– tigerjack89
Mar 28 at 6:41




$begingroup$
@Maxim thanks, it is surely better than nothing. But it could be great to also show the exact solution (constants throws in) or at least an approximation of it. Of course I'm also interested in the explanation (which in the case of master theorem is obvious).
$endgroup$
– tigerjack89
Mar 28 at 6:41










1 Answer
1






active

oldest

votes


















2





+50







$begingroup$

For the simplicity put $T(n)=6S(n)$ for each $n$. Then the sequence $S$ satisfies the same recurrence relation. A computed graph of the function $S$ suggests that it is piecewise linear with bends at $n=2^k+1$ and $2^k+2^k-1+1$. Indeed, by induction we can show that $S(2^k+1)=4^k-1$ and $S(2^k+2^k-1+1)=3cdot 4^k-1$ for each $kge 1$. Also by induction we can show that between these points $S$ is linear, that is $$S(2^k+lambda 2^k-1+1)=(1+2lambda)4^k-1$$ and $$S(2^k+2^k-1+lambda 2^k-1+1)=(3+lambda)4^k-1$$ for each integer $2^k-1lambda$ between $0$ and $2^k-1$. The found form of the function $S$ easily implies bounds $frac 14letfracS(n) (n-1)^2le frac 13$ (and $tfrac 32(n−1)^2le T(n)le 2(n−1)^2$), for each $nge 3$.






share|cite|improve this answer











$endgroup$








  • 1




    $begingroup$
    Sorry for the late response. Thanks for your help, I'm trying to understand your steps. Anyhow, how could one derive the asymptotic behaviour from this?
    $endgroup$
    – tigerjack89
    Apr 1 at 15:54










  • $begingroup$
    Giving therefore $frac32(n-1)^2 leq T(n) leq 2(n-1)^2$, am I right?
    $endgroup$
    – tigerjack89
    Apr 3 at 8:34











  • $begingroup$
    Well, I couldn't hope for a better bound!!! I'm still trying to figure out your method, but anyhow many thanks for your help. Would you add the last comments in your answer?
    $endgroup$
    – tigerjack89
    Apr 3 at 8:40










  • $begingroup$
    Thanks again for the help.
    $endgroup$
    – tigerjack89
    Apr 3 at 8:48











Your Answer








StackExchange.ready(function()
var channelOptions =
tags: "".split(" "),
id: "69"
;
initTagRenderer("".split(" "), "".split(" "), channelOptions);

StackExchange.using("externalEditor", function()
// Have to fire editor after snippets, if snippets enabled
if (StackExchange.settings.snippets.snippetsEnabled)
StackExchange.using("snippets", function()
createEditor();
);

else
createEditor();

);

function createEditor()
StackExchange.prepareEditor(
heartbeatType: 'answer',
autoActivateHeartbeat: false,
convertImagesToLinks: true,
noModals: true,
showLowRepImageUploadWarning: true,
reputationToPostImages: 10,
bindNavPrevention: true,
postfix: "",
imageUploader:
brandingHtml: "Powered by u003ca class="icon-imgur-white" href="https://imgur.com/"u003eu003c/au003e",
contentPolicyHtml: "User contributions licensed under u003ca href="https://creativecommons.org/licenses/by-sa/3.0/"u003ecc by-sa 3.0 with attribution requiredu003c/au003e u003ca href="https://stackoverflow.com/legal/content-policy"u003e(content policy)u003c/au003e",
allowUrls: true
,
noCode: true, onDemand: true,
discardSelector: ".discard-answer"
,immediatelyShowMarkdownHelp:true
);



);













draft saved

draft discarded


















StackExchange.ready(
function ()
StackExchange.openid.initPostLogin('.new-post-login', 'https%3a%2f%2fmath.stackexchange.com%2fquestions%2f3162321%2fsolve-recurrence-relation-for-non-trivial-base-case%23new-answer', 'question_page');

);

Post as a guest















Required, but never shown

























1 Answer
1






active

oldest

votes








1 Answer
1






active

oldest

votes









active

oldest

votes






active

oldest

votes









2





+50







$begingroup$

For the simplicity put $T(n)=6S(n)$ for each $n$. Then the sequence $S$ satisfies the same recurrence relation. A computed graph of the function $S$ suggests that it is piecewise linear with bends at $n=2^k+1$ and $2^k+2^k-1+1$. Indeed, by induction we can show that $S(2^k+1)=4^k-1$ and $S(2^k+2^k-1+1)=3cdot 4^k-1$ for each $kge 1$. Also by induction we can show that between these points $S$ is linear, that is $$S(2^k+lambda 2^k-1+1)=(1+2lambda)4^k-1$$ and $$S(2^k+2^k-1+lambda 2^k-1+1)=(3+lambda)4^k-1$$ for each integer $2^k-1lambda$ between $0$ and $2^k-1$. The found form of the function $S$ easily implies bounds $frac 14letfracS(n) (n-1)^2le frac 13$ (and $tfrac 32(n−1)^2le T(n)le 2(n−1)^2$), for each $nge 3$.






share|cite|improve this answer











$endgroup$








  • 1




    $begingroup$
    Sorry for the late response. Thanks for your help, I'm trying to understand your steps. Anyhow, how could one derive the asymptotic behaviour from this?
    $endgroup$
    – tigerjack89
    Apr 1 at 15:54










  • $begingroup$
    Giving therefore $frac32(n-1)^2 leq T(n) leq 2(n-1)^2$, am I right?
    $endgroup$
    – tigerjack89
    Apr 3 at 8:34











  • $begingroup$
    Well, I couldn't hope for a better bound!!! I'm still trying to figure out your method, but anyhow many thanks for your help. Would you add the last comments in your answer?
    $endgroup$
    – tigerjack89
    Apr 3 at 8:40










  • $begingroup$
    Thanks again for the help.
    $endgroup$
    – tigerjack89
    Apr 3 at 8:48















2





+50







$begingroup$

For the simplicity put $T(n)=6S(n)$ for each $n$. Then the sequence $S$ satisfies the same recurrence relation. A computed graph of the function $S$ suggests that it is piecewise linear with bends at $n=2^k+1$ and $2^k+2^k-1+1$. Indeed, by induction we can show that $S(2^k+1)=4^k-1$ and $S(2^k+2^k-1+1)=3cdot 4^k-1$ for each $kge 1$. Also by induction we can show that between these points $S$ is linear, that is $$S(2^k+lambda 2^k-1+1)=(1+2lambda)4^k-1$$ and $$S(2^k+2^k-1+lambda 2^k-1+1)=(3+lambda)4^k-1$$ for each integer $2^k-1lambda$ between $0$ and $2^k-1$. The found form of the function $S$ easily implies bounds $frac 14letfracS(n) (n-1)^2le frac 13$ (and $tfrac 32(n−1)^2le T(n)le 2(n−1)^2$), for each $nge 3$.






share|cite|improve this answer











$endgroup$








  • 1




    $begingroup$
    Sorry for the late response. Thanks for your help, I'm trying to understand your steps. Anyhow, how could one derive the asymptotic behaviour from this?
    $endgroup$
    – tigerjack89
    Apr 1 at 15:54










  • $begingroup$
    Giving therefore $frac32(n-1)^2 leq T(n) leq 2(n-1)^2$, am I right?
    $endgroup$
    – tigerjack89
    Apr 3 at 8:34











  • $begingroup$
    Well, I couldn't hope for a better bound!!! I'm still trying to figure out your method, but anyhow many thanks for your help. Would you add the last comments in your answer?
    $endgroup$
    – tigerjack89
    Apr 3 at 8:40










  • $begingroup$
    Thanks again for the help.
    $endgroup$
    – tigerjack89
    Apr 3 at 8:48













2





+50







2





+50



2




+50



$begingroup$

For the simplicity put $T(n)=6S(n)$ for each $n$. Then the sequence $S$ satisfies the same recurrence relation. A computed graph of the function $S$ suggests that it is piecewise linear with bends at $n=2^k+1$ and $2^k+2^k-1+1$. Indeed, by induction we can show that $S(2^k+1)=4^k-1$ and $S(2^k+2^k-1+1)=3cdot 4^k-1$ for each $kge 1$. Also by induction we can show that between these points $S$ is linear, that is $$S(2^k+lambda 2^k-1+1)=(1+2lambda)4^k-1$$ and $$S(2^k+2^k-1+lambda 2^k-1+1)=(3+lambda)4^k-1$$ for each integer $2^k-1lambda$ between $0$ and $2^k-1$. The found form of the function $S$ easily implies bounds $frac 14letfracS(n) (n-1)^2le frac 13$ (and $tfrac 32(n−1)^2le T(n)le 2(n−1)^2$), for each $nge 3$.






share|cite|improve this answer











$endgroup$



For the simplicity put $T(n)=6S(n)$ for each $n$. Then the sequence $S$ satisfies the same recurrence relation. A computed graph of the function $S$ suggests that it is piecewise linear with bends at $n=2^k+1$ and $2^k+2^k-1+1$. Indeed, by induction we can show that $S(2^k+1)=4^k-1$ and $S(2^k+2^k-1+1)=3cdot 4^k-1$ for each $kge 1$. Also by induction we can show that between these points $S$ is linear, that is $$S(2^k+lambda 2^k-1+1)=(1+2lambda)4^k-1$$ and $$S(2^k+2^k-1+lambda 2^k-1+1)=(3+lambda)4^k-1$$ for each integer $2^k-1lambda$ between $0$ and $2^k-1$. The found form of the function $S$ easily implies bounds $frac 14letfracS(n) (n-1)^2le frac 13$ (and $tfrac 32(n−1)^2le T(n)le 2(n−1)^2$), for each $nge 3$.







share|cite|improve this answer














share|cite|improve this answer



share|cite|improve this answer








edited Apr 3 at 8:46

























answered Mar 30 at 2:12









Alex RavskyAlex Ravsky

43.3k32583




43.3k32583







  • 1




    $begingroup$
    Sorry for the late response. Thanks for your help, I'm trying to understand your steps. Anyhow, how could one derive the asymptotic behaviour from this?
    $endgroup$
    – tigerjack89
    Apr 1 at 15:54










  • $begingroup$
    Giving therefore $frac32(n-1)^2 leq T(n) leq 2(n-1)^2$, am I right?
    $endgroup$
    – tigerjack89
    Apr 3 at 8:34











  • $begingroup$
    Well, I couldn't hope for a better bound!!! I'm still trying to figure out your method, but anyhow many thanks for your help. Would you add the last comments in your answer?
    $endgroup$
    – tigerjack89
    Apr 3 at 8:40










  • $begingroup$
    Thanks again for the help.
    $endgroup$
    – tigerjack89
    Apr 3 at 8:48












  • 1




    $begingroup$
    Sorry for the late response. Thanks for your help, I'm trying to understand your steps. Anyhow, how could one derive the asymptotic behaviour from this?
    $endgroup$
    – tigerjack89
    Apr 1 at 15:54










  • $begingroup$
    Giving therefore $frac32(n-1)^2 leq T(n) leq 2(n-1)^2$, am I right?
    $endgroup$
    – tigerjack89
    Apr 3 at 8:34











  • $begingroup$
    Well, I couldn't hope for a better bound!!! I'm still trying to figure out your method, but anyhow many thanks for your help. Would you add the last comments in your answer?
    $endgroup$
    – tigerjack89
    Apr 3 at 8:40










  • $begingroup$
    Thanks again for the help.
    $endgroup$
    – tigerjack89
    Apr 3 at 8:48







1




1




$begingroup$
Sorry for the late response. Thanks for your help, I'm trying to understand your steps. Anyhow, how could one derive the asymptotic behaviour from this?
$endgroup$
– tigerjack89
Apr 1 at 15:54




$begingroup$
Sorry for the late response. Thanks for your help, I'm trying to understand your steps. Anyhow, how could one derive the asymptotic behaviour from this?
$endgroup$
– tigerjack89
Apr 1 at 15:54












$begingroup$
Giving therefore $frac32(n-1)^2 leq T(n) leq 2(n-1)^2$, am I right?
$endgroup$
– tigerjack89
Apr 3 at 8:34





$begingroup$
Giving therefore $frac32(n-1)^2 leq T(n) leq 2(n-1)^2$, am I right?
$endgroup$
– tigerjack89
Apr 3 at 8:34













$begingroup$
Well, I couldn't hope for a better bound!!! I'm still trying to figure out your method, but anyhow many thanks for your help. Would you add the last comments in your answer?
$endgroup$
– tigerjack89
Apr 3 at 8:40




$begingroup$
Well, I couldn't hope for a better bound!!! I'm still trying to figure out your method, but anyhow many thanks for your help. Would you add the last comments in your answer?
$endgroup$
– tigerjack89
Apr 3 at 8:40












$begingroup$
Thanks again for the help.
$endgroup$
– tigerjack89
Apr 3 at 8:48




$begingroup$
Thanks again for the help.
$endgroup$
– tigerjack89
Apr 3 at 8:48

















draft saved

draft discarded
















































Thanks for contributing an answer to Mathematics Stack Exchange!


  • Please be sure to answer the question. Provide details and share your research!

But avoid


  • Asking for help, clarification, or responding to other answers.

  • Making statements based on opinion; back them up with references or personal experience.

Use MathJax to format equations. MathJax reference.


To learn more, see our tips on writing great answers.




draft saved


draft discarded














StackExchange.ready(
function ()
StackExchange.openid.initPostLogin('.new-post-login', 'https%3a%2f%2fmath.stackexchange.com%2fquestions%2f3162321%2fsolve-recurrence-relation-for-non-trivial-base-case%23new-answer', 'question_page');

);

Post as a guest















Required, but never shown





















































Required, but never shown














Required, but never shown












Required, but never shown







Required, but never shown

































Required, but never shown














Required, but never shown












Required, but never shown







Required, but never shown







Popular posts from this blog

Lowndes Grove History Architecture References Navigation menu32°48′6″N 79°57′58″W / 32.80167°N 79.96611°W / 32.80167; -79.9661132°48′6″N 79°57′58″W / 32.80167°N 79.96611°W / 32.80167; -79.9661178002500"National Register Information System"Historic houses of South Carolina"Lowndes Grove""+32° 48' 6.00", −79° 57' 58.00""Lowndes Grove, Charleston County (260 St. Margaret St., Charleston)""Lowndes Grove"The Charleston ExpositionIt Happened in South Carolina"Lowndes Grove (House), Saint Margaret Street & Sixth Avenue, Charleston, Charleston County, SC(Photographs)"Plantations of the Carolina Low Countrye

random experiment with two different functions on unit interval Announcing the arrival of Valued Associate #679: Cesar Manara Planned maintenance scheduled April 23, 2019 at 00:00UTC (8:00pm US/Eastern)Random variable and probability space notionsRandom Walk with EdgesFinding functions where the increase over a random interval is Poisson distributedNumber of days until dayCan an observed event in fact be of zero probability?Unit random processmodels of coins and uniform distributionHow to get the number of successes given $n$ trials , probability $P$ and a random variable $X$Absorbing Markov chain in a computer. Is “almost every” turned into always convergence in computer executions?Stopped random walk is not uniformly integrable

How should I support this large drywall patch? Planned maintenance scheduled April 23, 2019 at 00:00UTC (8:00pm US/Eastern) Announcing the arrival of Valued Associate #679: Cesar Manara Unicorn Meta Zoo #1: Why another podcast?How do I cover large gaps in drywall?How do I keep drywall around a patch from crumbling?Can I glue a second layer of drywall?How to patch long strip on drywall?Large drywall patch: how to avoid bulging seams?Drywall Mesh Patch vs. Bulge? To remove or not to remove?How to fix this drywall job?Prep drywall before backsplashWhat's the best way to fix this horrible drywall patch job?Drywall patching using 3M Patch Plus Primer